Đến nội dung

Hình ảnh

Tiếp sức bất đẳng thức


  • Please log in to reply
Chủ đề này có 223 trả lời

#181
Gachdptrai12

Gachdptrai12

    Thượng sĩ

  • Điều hành viên THCS
  • 280 Bài viết

Ta chỉ cần chứng minh

\[(a+b+c)^3 \geqslant 6\sqrt{3}|(a-b)(b-c)(c-a)|.\]

Giả sử $a \geqslant b \geqslant c.$ Áp dụng bất đẳng thức AM-GM, ta có
\[\begin{aligned} \left |(a-b)(b-c)(c-a)\right| &=(a-c)(b-c)(a-b)\\&\le (a+c)\cdot b \cdot (a+c-b)\\&=\frac{1}{2} \cdot \left (1+\sqrt{3} \right )(a+c)\cdot b \left ( -1+\sqrt{3} \right ) \cdot (a+c-b)\\& \le \frac{1}{2}\left [ \frac{\left (1+\sqrt{3} \right )(a+c)+ b \left ( -1+\sqrt{3} \right ) + (a+c-b)}{3} \right ]^3\\& =\frac{1}{6\sqrt{3}}(a+b+c)^3.\end{aligned}\]
Từ đó suy ra điều phải chứng minh.

bình phương => ta chứng minh bđt manh hơn (a-b)2(b-c)2(c-a)2$\leq \frac{27}{4}$ ta có$(b-c)^{2}\leq b^{2},(a-c)^{2}\leq a^{2}$

ta chứng minh$4a^{2}b^{2}(a-b)^{2}\leq 27$ áp dụng AM-GM ta có $4a^{2}b^{2}(a-b)^{2}\leq (\frac{2ab+2ab+(a-b)^{3}}{27})$ =$\frac{(a+b)^{6}}{27}$ mà a+b<=a+b+c=3 +. dpcm


Bài viết đã được chỉnh sửa nội dung bởi Gachdptrai12: 22-02-2016 - 10:43


#182
ineX

ineX

    Sĩ quan

  • Thành viên
  • 353 Bài viết

Từ một bất đẳng thức phụ cơ bản, tớ đề xuất bài này:

Bài 85:

Với a,b,c là các số thực dương có tích là 8. Tìm giá trị lớn nhất của biểu thức:

$P= \frac{1}{2a+b+6}+\frac{1}{2b+c+6}+\frac{1}{2c+a+6}$

 

 tiện thể hỏi luôn ai biết topic nào có toán bằng tiếng anh ko ạ


Bài viết đã được chỉnh sửa nội dung bởi ineX: 22-02-2016 - 22:11

"Tôi sinh ra là để thay đổi thế giới chứ không phải để thế giới thay đổi tôi" - Juliel

 

3cf67218ea144a6eb6caf571068071ff.1.gif


#183
Chris yang

Chris yang

    Thượng sĩ

  • Thành viên
  • 223 Bài viết

Từ một bất đẳng thức phụ cơ bản, tớ đề xuất bài này:

Bài 85:

Với a,b,c là các số thực dương có tích là 8. Tìm giá trị lớn nhất của biểu thức:

$P= \frac{1}{2a+b+6}+\frac{1}{2b+c+6}+\frac{1}{2c+a+6}$

 

 tiện thể hỏi luôn ai biết topic nào có toán bằng tiếng anh ko ạ

Cho $(a,b,c) \mapsto (2a^2,2b^2,2c^2)$. Ta sẽ đưa bài toán về CM $P=\sum \frac{1}{4a^2+2b^2+6}\leq \frac{1}{4}$ với điều kiện $abc=1$

Theo BĐT BCS dạng cộng mẫu $2P\leq  \frac{1}{4}\sum \left ( \frac{1}{a^2+b^2+1}+\frac{1}{a^2+2} \right )=\frac{1}{4}\left ( \sum\frac{1}{a^2+b^2+1} +\sum \frac{1}{a^2+2} \right )$

Do đó, cần có $ \sum\frac{1}{a^2+b^2+1} +\sum \frac{1}{a^2+2} =\leq 2$

Thực hiện biến đổi ngược dấu, bài toán tương đương $\sum \frac{2(a^2+b^2)}{a^2+b^2+1}+\sum\frac{a^2}{a^2+2}=M+N\geq 5$ $(\star)$

Không mất TQ, giả sử $a\geq b\geq c$

Áp dụng BĐT Cauchy Schwarz: 

$M=\sum \frac{(a+b)^2}{a^2+b^2+1}+\sum \frac{(a-b)^2}{a^2+b^2+1}\geq \frac{4(a+b+c)^2+4(a-c)^2}{2(a^2+b^2+c^2)+3}\geq 4$

$\Leftrightarrow (a+b+c)^2+(a-c)^2\geq 2(a^2+b^2+c^2)+3\Leftrightarrow 2ab+2bc\geq b^2+3$

Có $(a-b)(b-c)\geq 0\rightarrow ab+bc\geq b^2+ac\rightarrow 2(ab+bc)\geq b^2+ac+ab+bc\geq b^2+3$ ( đúng vì $ab+bc+ac\geq 3$)

 

Do đó $M\geq 4$ $(1)$

Lại có $N=\sum \frac{a^2}{a^2+3}\geq \frac{(a+b+c)^2}{a^2+b^2+c^2+6}\geq 1\Leftrightarrow ab+bc+ac\geq 3$ ( đúng) $(2)$

Từ $(1)$ và $(2)$ suy ra $M+N\geq 5$, hay BĐT $(\star)$ đúng. Suy ra $P\leq \frac{1}{4}$

Dấu $=$ xảy ra khi ba biến bằng nhau

 

------------------------------------------------------------------------------------------------------------

Một số trang web toán bằng tiếng anh bạn có thể tham khảo là mathlinks.ro, math.stackexchange.com, brilliant.org



#184
cyndaquil

cyndaquil

    Hạ sĩ

  • Thành viên
  • 63 Bài viết

Bài 86

Cho $a,b,c>0$ thõa mãn $a+b+c=2$. C/m : $\sum \frac{bc}{20+3a^2} \le \frac{1}{16} $


Bài viết đã được chỉnh sửa nội dung bởi cyndaquil: 23-02-2016 - 18:11


#185
Gachdptrai12

Gachdptrai12

    Thượng sĩ

  • Điều hành viên THCS
  • 280 Bài viết

Bài 86:cho $x,y$ là các số thực thỏa $x+y+2=2(\sqrt{x-1} + \sqrt{y+2})$. Tìm min $\frac{1}{x}+\frac{1}

baì này mình làm thế này ko biết đúng hay không thấy đăng mà không ai giải 

áp dụng bđt C-S ta có$2(\sqrt{x-1}+\sqrt{y+2})\geq 2\sqrt{2(x+y+1)}$

đặt x+y+1=t ta có từ giả thuyêt=>t+1$\geq 2\sqrt{2(t)}$ ta chăn được t rồi ta có 

$\frac{1}{x}+\frac{1}{y+3}\geq \frac{4}{x+y+3}=\frac{1}{t+2}$ => ra được min mà vừa đảm bảo được dấu bằng



#186
leanh9adst

leanh9adst

    Thượng sĩ

  • Thành viên
  • 213 Bài viết

Bài 87:Cho $a,b,c>0$ và $a+b+c=3$.CMR:$(a+b)(b+c)(c+a)\geq (ab+c)(bc+a)(ca+b)$

Bài 88: Cho $a,b,c$ là các số dương.CMR:

$\frac{a^2}{2a^2+(b+c-a)^2}+\frac{b^2}{2b^2+(c+a-b)^2}+\frac{c^2}{2c^2+(a+b-c)^2}\leq 1$

Bài 89: Cho các số thực a,b,c thỏa mãn $a^2+b^2+c^2=2(ab+bc+ca)$ và $abc\neq 0$.CMR:

$\sum \frac{\left | a-b \right |}{\sqrt{2ab+c^2}}\geq 2$

P/s: Dùng Cô-si nhé mọi người!


Mặt trời mọc rồi lặn,mặt trăng tròn rồi lại khuyết nhưng ánh sáng mà người thầy rọi vào ta sẽ còn mãi trong cuộc đời!


#187
Chris yang

Chris yang

    Thượng sĩ

  • Thành viên
  • 223 Bài viết

Bài 86

Cho $a,b,c>0$ thõa mãn $a+b+c=2$. C/m : $\sum \frac{bc}{20+3a^2} \le \frac{1}{16} $

Áp dụng BĐT Cauchy Schwarz:

$P=\sum \frac{bc}{3a^2+5(a+b+c)^2}\leq \sum \frac{bc}{2a(a+b+c)+\frac{14(a+b+c)^2}{3}}=\frac{3}{4}\sum \frac{bc}{10a+7b+7c}$

$P\leq \frac{3}{4.12^2}\sum \left ( \frac{5bc}{a+b}+\frac{5bc}{a+c}+\frac{2bc}{b+c} \right )=\frac{3}{4.12^2}\left ( 5(a+b+c)+\frac{2ab}{a+b}+\frac{2bc}{b+c}+\frac{2ac}{a+c} \right )\leq \frac{3.6}{4.12^2}(a+b+c)=\frac{1}{16}$

Dấu $=$ xảy ra khi ba biến bằng nhau

 

Bài 87:Cho $a,b,c>0$ và $a+b+c=3$.CMR:$(a+b)(b+c)(c+a)\geq (ab+c)(bc+a)(ca+b)$

Bài 88: Cho $a,b,c$ là các số dương.CMR:

$\frac{a^2}{2a^2+(b+c-a)^2}+\frac{b^2}{2b^2+(c+a-b)^2}+\frac{c^2}{2c^2+(a+b-c)^2}\leq 1$

 

Bài 87. Áp dụng BĐT AM GM ngược dấu:

$(ab+c)(bc+a)\leq \frac{(ab+c+bc+a)^2}{4}=\frac{(a+c)^2(b+1)^2}{4}$. Tương tự với các biểu thức còn lại, suy ra:

$VP^2\leq \frac{[(a+b)(b+c)(c+a)]^2(a+1)(b+1)(c+1)}{64}$. Áp dụng AM-GM lần nữa có $(a+1)(b+1)(c+1)\leq \left ( \frac{a+b+c+3}{3} \right )^3=8$

$\Rightarrow VP^2\leq (a+b)^2(b+c)^2(c+a)^2\Rightarrow (ab+c)(bc+a)(ca+b)\leq (a+b)(b+c)(c+a)$ ( đpcm)

 

Bài 88:

Đặt $(b+c-a,c+a-b,a+b-c)\mapsto (x,y,z)\Rightarrow (a,b,c)=(\frac{y+z}{2},\frac{x+z}{2}+\frac{x+y}{2}) $

Khi đó, thực hiện biến đổi ngược dấu,  BĐT cần chứng minh trở thành: $\sum \frac{x^2}{2x^2+(y+z)^2}\geq \frac{1}{2}$

BĐT này luôn đúng vì theo AM-GM ta có $\sum \frac{x^2}{2x^2+(y+z)^2}\geq \frac{x^2}{2x^2+2(y^2+z^2)}=\frac{1}{2}$ ( đpcm)


Bài viết đã được chỉnh sửa nội dung bởi ngocanh99: 25-02-2016 - 18:34


#188
ineX

ineX

    Sĩ quan

  • Thành viên
  • 353 Bài viết

Bài 90:

Với các số thực không âm x,y,z có tổng bằng 3. Chứng minh rằng:

$\frac{x}{x^{2}+y^{3}}+\frac{y}{y^{2}+z^{3}}+\frac{z}{z^{2}+x^{3}}$


"Tôi sinh ra là để thay đổi thế giới chứ không phải để thế giới thay đổi tôi" - Juliel

 

3cf67218ea144a6eb6caf571068071ff.1.gif


#189
Nguyenhuyen_AG

Nguyenhuyen_AG

    Trung úy

  • Thành viên nổi bật 2016
  • 945 Bài viết

Bài 88: Cho $a,b,c$ là các số dương.CMR:

$$\frac{a^2}{2a^2+(b+c-a)^2}+\frac{b^2}{2b^2+(c+a-b)^2}+\frac{c^2}{2c^2+(a+b-c)^2}\leq 1$$

 

Ta viết bất đẳng thức cần chứng minh lại như sau

\[\frac{(b+c-a)^2}{2a^2+(b+c-a)^2}+\frac{(c+a-b)^2}{2b^2+(c+a-b)^2}+\frac{(a+b-c)^2}{2c^2+(a+b-c)^2} \geqslant 1.\]

Áp dụng bất đẳng thức Cauchy-Schwarz, ta có

\[\sum \frac{(b+c-a)^2}{2a^2+(b+c-a)^2} \geqslant \frac{\left [ \displaystyle \sum (b+c-a)^2 \right ]^2}{\displaystyle \sum \left [ 2a^2(b+c-a)^2+(b+c-a)^4 \right ]}.\]

Như vậy ta cần chứng minh

\[\left [ \displaystyle \sum (b+c-a)^2 \right ]^2 \geqslant \sum \left [ 2a^2(b+c-a)^2+(b+c-a)^4 \right ],\]

khai triển và thu gọn thành

\[a^4+b^4+c^4+abc(a+b+c) \geqslant \sum ab(a^2+b^2).\]

Hiển nhiên đúng theo bất đẳng thức Schur bậc 4.


Nguyen Van Huyen
Ho Chi Minh City University Of Transport

#190
hangdiemdieuhoa1999

hangdiemdieuhoa1999

    Binh nhì

  • Thành viên mới
  • 12 Bài viết

ai giúp bài này với

BÀI 91:

với a,b,c dương. a+b+c=10.

min:

$M= \frac{a^{2}-a}{9}+\frac{b^{2}-b}{5}+\frac{c^{2}-c}{3}$



#191
hangdiemdieuhoa1999

hangdiemdieuhoa1999

    Binh nhì

  • Thành viên mới
  • 12 Bài viết

BÀI 92:CHO a,b,c dương abc=1.

CMR:

$\sum \frac{b+c}{\sqrt{a}}\geq \sum \sqrt{a}+3$



#192
ineX

ineX

    Sĩ quan

  • Thành viên
  • 353 Bài viết

BÀI 92:CHO a,b,c dương abc=1.

CMR:

$\sum \frac{b+c}{\sqrt{a}}\geq \sum \sqrt{a}+3$

Bài này có nhiều phương án giải quyết. Sau đây là cách của mình:

Với các số thực dương a,b,c ta có bất đẳng thức AM-GM:  $\frac{b+c}{\sqrt{a}}\geq \frac{2\sqrt{bc}}{\sqrt{a}}$

Làm tương tự và cộng ba bất đẳng thức lại, ta phải chứng mình:

$\sum \frac{2\sqrt{bc}}{\sqrt{a}}\geq \sum \sqrt{a}+3\Leftrightarrow 2\sum ab\geq \sqrt{abc}(\sum a)+3\sqrt{abc}\Leftrightarrow 2\sum ab\geq \sqrt{abc}(\sum \sqrt{a})+3$

Mặt khác, theo bất đẳng thức Cauchy- Schwarz, ta lại có:

$\sum ab\geq \sqrt{abc}\left ( \sum \sqrt{a} \right )$

Và theo bđt AM-GM ta có:

$\sum ab\geq \sqrt[3]{a^{2}b^{2}c^{2}}=1$

Như vậy, ta có điều phải chứng minh!


"Tôi sinh ra là để thay đổi thế giới chứ không phải để thế giới thay đổi tôi" - Juliel

 

3cf67218ea144a6eb6caf571068071ff.1.gif


#193
superpower

superpower

    Sĩ quan

  • Thành viên
  • 492 Bài viết

ai giúp bài này với

BÀI 91:

với a,b,c dương. a+b+c=10.

min:

$M= \frac{a^{2}-a}{9}+\frac{b^{2}-b}{5}+\frac{c^{2}-c}{3}$

Chứng minh bất đẳng thức phụ sau

$\frac{a^2-a}{9} \geq \frac{17}{27}a -\frac{100}{81} <=> (3a-10)^2 \geq 0 $

Do đó cộng các bất đẳng thức lại, ta có 

$M \geq \frac{17}{27} . 10 -\frac{100}{27} =\frac{70}{27} $



#194
ineX

ineX

    Sĩ quan

  • Thành viên
  • 353 Bài viết

Topic này bỏ hoang rồi ư?

Bài 93:

Với các số thực dương x,y,z thỏa mãn tổng là 1. Tìm giá trị nhỏ nhất của:

$P= \sqrt{\frac{1-x}{1+x}}+\sqrt{\frac{1-y}{1+y}}+\sqrt{\frac{1-z}{1+z}}$


"Tôi sinh ra là để thay đổi thế giới chứ không phải để thế giới thay đổi tôi" - Juliel

 

3cf67218ea144a6eb6caf571068071ff.1.gif


#195
minhhien2001

minhhien2001

    Trung sĩ

  • Thành viên
  • 177 Bài viết

Bài 94:Cho $a,b,c>0$ . $a^2+b^2+c^2=1$ Chứng minh: $\frac{a^2}{1+b-a}+\frac{b^2}{1+c-b}+\frac{c^2}{1+a-c}\geqslant 1$


Bài viết đã được chỉnh sửa nội dung bởi royal1534: 18-03-2016 - 21:59


#196
tranductucr1

tranductucr1

    Thượng sĩ

  • Thành viên
  • 208 Bài viết

Từ một bất đẳng thức phụ cơ bản, tớ đề xuất bài này:

Bài 85:

Với a,b,c là các số thực dương có tích là 8. Tìm giá trị lớn nhất của biểu thức:

$P= \frac{1}{2a+b+6}+\frac{1}{2b+c+6}+\frac{1}{2c+a+6}$

 

 tiện thể hỏi luôn ai biết topic nào có toán bằng tiếng anh ko ạ

đổi biến (a,b,c)~($\frac{2x}{y},\frac{2y}{z},\frac{2z}{x}$)
=>$ P= \sum \frac{yz}{4xz+2y^2+6yz} $
$3-6P=\sum \frac{4xz+2y^2}{4xz+2y^2+6yz}$

ta có $\sum \frac{4xz}{4xz+6yz+2y^2} \geq \frac{4(xy+xz+yz)^2}{4((xz)^2+(yz)^2+(xy)^2) +8xyz (x+y+z)} =1 $

và     $ \sum \frac{2y^2}{4xz+2y^2+6xz} \geq \frac{2(x+z+z)^2}{2(x^2+y^2+z^2)+10(xz+yz+xy)} = \frac{(x+y+z)^2}{(x+y+z)^2+3(xz+yz+xy)} \geq \frac{(x+y+z)^2}{(x+y+z)^2+3*\frac{(x+y+z)^2}{3}} =\frac{1}{1+1}=\frac{1}{2}$
Vậy $3-6P \geq \frac{1}{2}+1$ => $P \leq \frac{1}{4}$


Để trở thành người phi thường, tôi không cho phép bản thân tầm thường

Roronoa Zoro- One piece

Liên lạc với tôi qua https://www.facebook...0010200906065  


#197
Hoang Duong

Hoang Duong

    Binh nhất

  • Thành viên
  • 27 Bài viết

Cho a,b,c>0 C/m: $\frac{a^2}{1+b-a}+\frac{b^2}{1+c-b}+\frac{c^2}{1+a-c}\geqslant 1$

Mình chắc là phải có giả thiết $a^2+b^2+c^2=1$ hoặc tương tự thì mới ổn
ta có:

$\sum c^3=\sum\frac{a^3+a^3+b^3}{3}\geq \sum a^2b$

Áp dụng:

$\sum\frac{a^2}{1+b-a}=\frac{a^4}{a^2+a^2b-a^3}\geq\sum\frac{(\sum a^2)}{\sum a^2+\sum a^2b-\sum a^3}\geq\sum a^2=1$


"Và tôi vẫn còn yêu em..."


#198
Hoang Duong

Hoang Duong

    Binh nhất

  • Thành viên
  • 27 Bài viết

đổi biến (a,b,c)~($\frac{2x}{y},\frac{2y}{z},\frac{2z}{x}$)
=>$ P= \sum \frac{yz}{4xz+2y^2+6yz} $
$3-6P=\sum \frac{4xz+2y^2}{4xz+2y^2+6yz}$

ta có $\sum \frac{4xz}{4xz+6yz+2y^2} \geq \frac{4(xy+xz+yz)^2}{4((xz)^2+(yz)^2+(xy)^2) +8xyz (x+y+z)} =1 $

và     $ \sum \frac{2y^2}{4xz+2y^2+6xz} \geq \frac{2(x+z+z)^2}{2(x^2+y^2+z^2)+10(xz+yz+xy)} = \frac{(x+y+z)^2}{(x+y+z)^2+3(xz+yz+xy)} \geq \frac{(x+y+z)^2}{(x+y+z)^2+3*\frac{(x+y+z)^2}{3}} =\frac{1}{1+1}=\frac{1}{2}$
Vậy $3-6P \geq \frac{1}{2}+1$ => $P \leq \frac{1}{4}$

Lời giải khá hay
nhưng có thể hơi khó hiểu,

mình xin trình bày lời giải khác:

đổi biến: $(x,y,z)\rightarrow (2a^2,2b^2,2c^2)\Rightarrow abc=1$

khi đó bất đẳng thức trở thành:

$\sum\frac{1}{4a^2+2b^2+6}=\sum\frac{1}{2a^2+2b^2+2a^2+2+4}\leq\sum\frac{1}{4ab+4a+4}=\frac{1}{4}\sum\frac{1}{ab+a+1}=1/4$

Công việc chứng minh $\sum\frac{1}{ab+a+1}=1$ với $abc=1$ khá đơn giản, bạn chỉ cần rút thế hợp lí là được


"Và tôi vẫn còn yêu em..."


#199
tranductucr1

tranductucr1

    Thượng sĩ

  • Thành viên
  • 208 Bài viết

bài bất mới 
Bài 95:cho a,b,c là các số thực dương

$ab^2+bc^2+ca^2=3$ 
chứng minh rằng $\frac{2a^5+3b^5}{ab}+\frac{2b^5+3c^5}{bc}+\frac{2c^5+3a^5}{ac} \geq 15(a^3+b^3+c^3-2)$ 
----------- Đề Thanh Hóa--- nguồn : facebook 
ps topic hay mong nó tiếp tục phát triển  >:)  >:)  >:)  >:)  >:)  >:)


Bài viết đã được chỉnh sửa nội dung bởi HappyLife: 16-03-2016 - 18:41

Để trở thành người phi thường, tôi không cho phép bản thân tầm thường

Roronoa Zoro- One piece

Liên lạc với tôi qua https://www.facebook...0010200906065  


#200
the unknown

the unknown

    Thượng sĩ

  • Thành viên
  • 208 Bài viết

Em xin gửi hai bài ạ:

Bài 96: Cho $a,b,c,d$ là các số thực thỏa mãn điều kiện:

                                     $abc+bcd+cda+dab=a+b+c+d+\sqrt{2016}$.

Chứng minh rằng: $(a^2+1)(b^2+1)(c^2+1)(d^2+1)\geq 2016$.

Bài 97: Cho $a,b,c$ là các số thực dương thỏa mãn:

                                      $\frac{32}{a+32}+\frac{3}{2b+3}\leq \frac{4c}{4c+21}$.

Tìm giá trị nhỏ nhất của $abc$.


Bài viết đã được chỉnh sửa nội dung bởi HappyLife: 16-03-2016 - 18:42

$\texttt{If you don't know where you are going, any road will get you there}$





1 người đang xem chủ đề

0 thành viên, 1 khách, 0 thành viên ẩn danh